Mathcenter Forum

Mathcenter Forum (https://www.mathcenter.net/forum/index.php)
-   Calculus and Analysis (https://www.mathcenter.net/forum/forumdisplay.php?f=27)
-   -   Real analysis problem (https://www.mathcenter.net/forum/showthread.php?t=2433)

M@gpie 11 เมษายน 2007 20:50

Real analysis problem
 
ขออนุญาตยึดกระทู้นี้ไว้ถาม-ตอบปัญหา Real analysis ที่ผมคิดไว้แต่ไม่รู้ว่า ถูกไหม แล้วกันนะครับ เอามาถามพร้อมรบกวนพี่ๆ ตรวจเช็คความถูกต้อง เช่นเคยครับ

คำถามรวมๆหลายๆเรื่องนะครับ ท่านใดมีปัญหาน่าสนใจก็โพสไว้ได้เลยครับ :)

1. ให้ $A_n = [n,\infty )$ จงแสดงว่า $\bigcap_{n=1}^{\infty}A_n = \phi $

พิสูจน์ : สมมติ $\bigcap_{n=1}^{\infty}A_n \neq \phi $ จะได้ว่ามีจำนวนจริง $x$ ที่เป็นสมาชิกของ $A_n$ ทุก $n\in N$ แต่ไม่ว่าเราเลือกจำนวนจริง $x$ ใดก็ตามจะมี $n_0$ ที่ $x\notin A_{n_0}$ เสมอ ซึ่งขัดแย้ง #

2. ให้ $f:X\rightarrow [0,\infty ] $ เป็นฟังก์ชันที่หาเมเชอร์ได้ และ $A\in \mathcal{M}$ จงพิสูจน์ว่า

2.1 ถ้า $\int_{A}f d\mu = 0$ แล้วจะได้ว่า $f(x)=0\; a.e.$ on $A$

พิสูจน์ : ให้ $A_n = \{ x\in A : f(x) > \frac{1}{n} \}$ สำหรับทุกจำนวนนับ $n\in N$ จะเห็นว่า $A_1 \subset A_2 \subset A_3 \subset ... \subset A_n $ และ $\{ x \in A : f(x)>0 \} = \bigcup_{n=1}^{\infty} A_n $ ซึ่งโดยสมบัติของเมเชอร์จะได้ว่า ${\displaystyle \mu (\bigcup_{n=1}^{\infty}A_n) = \lim_{n\rightarrow \infty}\mu (A_n) } $
ต่อไปพิจารณา \[ \frac{1}{n}\mu (A_n) < \int_{A_n} f d\mu \leq \int_{A} f d\mu = 0, \; \; \forall n \in N \] ซึ่งจะได้ว่า $\mu (\{ x \in A : f(x)>0 \} ) = 0$ จึงได้ว่า $f(x)=0\; a.e.$ ตามต้องการ #

2.2 ถ้า $\int_{A}f d\mu <\infty $ แล้วจะได้ว่า $f(x) < \infty \; a.e.$ on $A$

พิสูจน์ : สมมติให้ $A_n =\{ x \in A : f(x) > n \}$ สำหรับทุกจำนวนนับ $n\in N$ จะเห็นว่า $A_1 \supset A_2 \supset ... \supset A_n$ และ $\{ x\in A : f(x)=\infty \} = \bigcap_{n=1}^{\infty}A_n $ โดยสมบัติของเมเชอร์จะได้ว่า ${\displaystyle \mu (\bigcap_{n=1}^{\infty}A_n) = \lim_{n\rightarrow \infty}\mu (A_n) } $
ต่อไปพิจารณา \[ n\mu (A_n) < \int_{A_n} f d\mu \leq \int_{A} f d\mu \leq M , \; \; \forall n \in N \] ซึ่งจะได้ว่า $\mu (\{ x\in A : f(x)=\infty \}) = \lim_{n\rightarrow \infty} \mu (A_n) \leq \lim_{n\rightarrow \infty}\frac{M}{n} = 0 $ ดังนั้น $\mu (A) = 0$ ทำให้ได้ว่า $f(x)< \infty \; a.e.$ ตามต้องการ #

3. ให้ $f\in L_1(R)$ จงแสดงว่า ${\displaystyle F(x) = \int_{-\infty}^{x} f(t)dt }$ เป็นฟังก์ชันต่อเนื่อง
พิสูจน์ : ให้ลำดับของจำนวนจริงที่ลู่เข้า $x_n\rightarrow x$ ต่อไปจะแสดงว่า $F(x_n)\rightarrow F(x)$
พิจารณา
\[ \lim_{n\rightarrow \infty} \mid \int_{-\infty}^{x_n} f(t)dt -\int_{-\infty}^{x} f(t)dt \mid = \mid \lim_{n\rightarrow \infty} \int_{x_n}^{x}f(x) dx \mid \]
พิจารณา \[ \lim_{n\rightarrow \infty}\int_{x_n}^{x}f(x) dx = \lim_{n\rightarrow \infty}\int_{-\infty}^{\infty}f(x)\chi_{[x,x_n]} dx\]
โดย จะเห็นว่า $|f(x)\chi_{[x,x_n]}|\leq |f(x)| \in L_1$ โดย Lebesgue Dominated Convergence Theorem จะได้ว่า \[ \lim_{n\rightarrow \infty}\int_{-\infty}^{\infty}f(x)\chi_{[x,x_n]} dx = \int_{-\infty}^{\infty}\lim_{n\rightarrow \infty}f(x)\chi_{[x,x_n]} dx = 0 \]
ดังนั้น $F$ ต่อเนื่อง

4. จงหาค่าของ \[ \lim_{n\rightarrow \infty} \int_0^{n} (1-\frac{x}{n})^ne^{x} dx\]
คิดไม่ออกครับ ว่าใช้วิธีไหนดี ?

ยังมีอีกครับแต่ยังคิดไม่ออก ไว้คิดออกแล้วจะมาโพสใหม่นะครับ :rolleyes:

TOP 11 เมษายน 2007 23:05

อ้างอิง:

ข้อความเดิมเขียนโดยคุณ M@gpie (ข้อความที่ 17740)
1. ให้ $A_n = [n,\infty )$ จงแสดงว่า $\bigcap_{n=1}^{\infty}A_n = \phi $

พิสูจน์ : สมมติ $\bigcap_{n=1}^{\infty}A_n \neq \phi $ จะได้ว่ามีจำนวนจริง $x$ ที่เป็นสมาชิกของ $A_n$ ทุก $n\in N$ แต่ไม่ว่าเราเลือกจำนวนจริง $x$ ใดก็ตามจะมี $n_0$ ที่ $x\notin A_{n_0}$ เสมอ ซึ่งขัดแย้ง #

ควรจะบอกไปเลยนะครับว่า สำหรับทุกจำนวนจริง $x \geq 0$ จะมี $n_0 = \lceil x \rceil + 1$ ที่ทำให้ $x\notin A_{n_0}$ ซึ่งทำให้เกิดข้อขัดแย้ง

M@gpie 11 เมษายน 2007 23:22

อ๋อ ขอบคุณพี่ TOP ครับ :D ผมรู้ว่ามีครับ แต่ไม่รู้จะเขียนยังไง แหะๆ

nooonuii 12 เมษายน 2007 12:34

ข้อ 1 อ้าง Archimedean Property ได้ครับ

ข้อ 4 แน่ใจเหรอครับว่าโจทย์เป็นแบบนี้จริงๆ

M@gpie 12 เมษายน 2007 19:14

ข้อ 4. โจทย์เป็นแบบนี้จริงครับ ก็ไม่รู้ว่าใช้ทฤษฎีไหน? หรือว่าโจทย์ผิด !!!

passer-by 12 เมษายน 2007 20:29

Simplify รอบที่ 1

$$ \int_0^n (1-\frac{x}{n})^n e^x \,\, dx = \frac{e^n}{n^n} \int_0^n x^n e^{-x} \,\, dx$$

Simplify รอบที่ 2 ถ้าผมไม่คิดเลขผิดนะครับ ค่าที่ถูก take limit คือ

$$ \frac{n!e^n}{n^n} -\frac{1}{n^{2n}}\sum_{k=0}^n \binom{n}{k}k! \big(\frac{1}{n^k}\big ) $$

Simplify รอบที่ 3 (ขอบคุณคุณ Warut ที่ตรวจทานให้ครับ) ค่าที่ถูก take limit คือ

$$ \frac{n!e^n}{n^n} -\sum_{k=0}^n \binom{n}{k}k! \big(\frac{1}{n^k}\big ) $$

nooonuii 13 เมษายน 2007 09:47

อ้างอิง:

ข้อความเดิมเขียนโดยคุณ M@gpie (ข้อความที่ 17768)
ข้อ 4. โจทย์เป็นแบบนี้จริงครับ ก็ไม่รู้ว่าใช้ทฤษฎีไหน? หรือว่าโจทย์ผิด !!!

โจทย์ไม่น่าจะผิดครับ แต่ลิมิตของมันนี่สิ เข้าหา $\infty$ รึเปล่า :unsure:

warut 13 เมษายน 2007 22:05

จากที่ผมลองแทนค่า $n=2$ คิดว่าการ simplify ครั้งที่ 2 ของคุณ passer-by น่าจะผิดนะครับ คำตอบที่ถูกน่าจะี่ไม่มี $\dfrac{1}{n^{2n}}$ แต่ผมก็ยังไม่รู้เลยว่ามันมาได้ยังไง

M@gpie 13 เมษายน 2007 22:38

พี่ passer-by เปลี่ยนตัวแปรเหรอครับ ทำไม มีอนุกรมมาด้วย แหะๆ น่ากลัว

passer-by 14 เมษายน 2007 05:21

ผม edit คำตอบตามข้อทักท้วงของคุณ Warut แล้วนะครับ

อ้างอิง:

ข้อความเดิมเขียนโดยคุณ M@gpie (ข้อความที่ 17799)
พี่ passer-by เปลี่ยนตัวแปรเหรอครับ ทำไม มีอนุกรมมาด้วย แหะๆ น่ากลัว

ไม่ได้เปลี่ยนตัวแปรหรอกครับ ก็ induction แล้วสังเกต pattern ของ $ \int_0^n x^ne^{-x} \,\, dx$ ธรรมดา นี่แหละครับ (แต่ผมไม่รู้เหมือนกันว่า มีแบบที่สั้นกว่านี้มั้ย)

warut 14 เมษายน 2007 07:33

จากที่คุณ passer-by หาไว้ว่า $$ \int_0^n (1-\frac{x}{n})^n e^x \, dx = \frac{n!e^n}{n^n} -\sum_{k=0}^n \binom{n}{k}k! \left( \frac{1}{n^k} \right) $$ และเนื่องจาก $$ \sum_{k=0}^n \binom{n}{k}k! \left( \frac{1}{n^k} \right) = \frac{n!}{n^n} \sum_{k=0}^n \frac{n^{n-k}}{(n-k)!} = \frac{n!}{n^n} \sum_{k=0}^n \frac{n^k}{k!} $$ ดังนั้น $$ \int_0^n (1-\frac{x}{n})^n e^x \, dx = \frac{n!}{n^n} \left( e^n - \sum_{k=0}^n \frac{n^k}{k!} \right) $$ $$= \frac{n!}{n^n} \left( \frac{n^{n+1}}{(n+1)!} + \frac{n^{n+2}}{(n+2)!} + \cdots \right) $$ $$= \frac{n}{n+1} + \frac{n^2}{(n+1)(n+2)} + \cdots $$ แต่ผมไม่แน่ใจเท่าไหร่ว่า หนทางนี้จะนำไปสู่คำตอบได้นะครับ

Punk 15 เมษายน 2007 10:19

ข้อ 4 ได้ อนันต์อย่างที่ nooonui ว่าแหละครับ เพราะว่า
\[
\lim_{n\to\infty}\left(1-\frac{x}{n}\right)^n=e^{-x}
\]
และใช้ Fatou's lemma (http://mathworld.wolfram.com/FatousLemma.html) กับลำดับ $f_n(x)=(1-x/n)^ne^x\chi_{[0,n]}$ ครับ

ถ้าจำไม่ผิด ข้อนี้คุ้นๆ อยู่นา...

M@gpie 15 เมษายน 2007 11:47

อ่า ตอนแรกผมก็ใช้ Fatou's Lemma เหมือนกันครับ ก็ได้คำตอบเป็น อนันต์แต่ผมนึกว่าจะมีวิธีได้คำตอบอื่น :D
แล้วข้อ 2.1, 2.2, 3 ล่ะครับ มีปัญหาอะไรรึเปล่าครับ ?

ขอบคุณพี่ๆทุกคนสำหรับทุกๆความเห็นครับ
ป.ล. ทำไมสมการในความเห็นของพี่ warut แสดงผลไม่สมบูรณ์ล่ะครับ ? ผมอ่านไม่ออก

M@gpie 16 เมษายน 2007 01:20

โพสเพิ่มครับ
5. Let $f_n : X\rightarrow [0,\infty ]$ be a sequence of measurable functions such that $f_1 \geq f_2 \geq ... \geq 0, \; \; f_n(x)\rightarrow f(x), \forall x \in X$ and ${\displaystyle \int_X f_1 d\mu < \infty}$. Prove that \[ \lim_{n\rightarrow \infty} \int_{X} f_n d\mu = \int_X f d\mu\]
Proof : Since $\mid f_n(x) \mid \leq f_1(x),\; $ and $f_n\rightarrow f, \; \; \forall x\in X$.
By Lebesgue Dominated Convergence Theorem (LDCT), we can change order of integration and limitation.
\[ \lim_{n\rightarrow \infty} \int_{X} f_n d\mu = \int_X f d\mu\] มีวิธีไม่ใช้ LDCT ไหมครับ ?

6. Suppose $f_n:X \rightarrow [0,\infty ]$ is measurable for each $n\in \mathbb{N}, \; \; f_n\rightarrow f$ pointwise, and ${\displaystyle \int_X f d\mu = \lim_{n\rightarrow \infty}\int_X f_n d\mu < \infty }$. Prove that \[\int_E f d\mu = \lim_{n\rightarrow \infty} \int_E f_n d\mu, \; \; \; \; \forall E \in \mathcal{M}\]

warut 16 เมษายน 2007 01:46

อ้างอิง:

ข้อความเดิมเขียนโดยคุณ M@gpie (ข้อความที่ 17823)
ป.ล. ทำไมสมการในความเห็นของพี่ warut แสดงผลไม่สมบูรณ์ล่ะครับ ? ผมอ่านไม่ออก

ถ้าใช้ IE มันจะมีปัญหาเยอะมาก ลองเปลี่ยนมาใช้ Firefox สิครับน่าจะช่วยได้


เวลาที่แสดงทั้งหมด เป็นเวลาที่ประเทศไทย (GMT +7) ขณะนี้เป็นเวลา 22:06

Powered by vBulletin® Copyright ©2000 - 2024, Jelsoft Enterprises Ltd.
Modified by Jetsada Karnpracha